Foros-FIUBA Foros HostingPortal
 FAQ  •  Buscar  •  Wiki  •  Apuntes  •  Planet  •  Mapa  •  Eyeon  •  Chat
Preferencias  •  Grupos de Usuarios
Registrarse  •  Perfil  •  Entrá para ver tus mensajes privados  •  Login
Ver tema siguiente
Ver tema anterior

Responder al tema Ver tema anteriorEnviar por mail a un amigo.Mostrar una Lista de los Usuarios que vieron este TemaGuardar este Tema como un archivoPrintable versionEntrá para ver tus mensajes privadosVer tema siguiente
Autor Mensaje
koreano
Nivel 9



Registrado: 15 Jul 2010
Mensajes: 1796

Carrera: No especificada
blank.gif
MensajePublicado: Mie Nov 23, 2011 5:25 pm  Asunto:  (Sin Asunto) Responder citandoFin de la PáginaVolver arriba

Hmm, no me cierra. El campo B es independiente de la curva que tomes porque te queda en función de la densidad de vueltas por metro (en SI).

Para mí el tema va por el lado de que el flujo concatenado también debe tener que ver con la cantidad de vueltas por unidad de longitud. El tema es que entonces la inducción mutua te queda:

[tex]M = \frac{\mu_0 N_1 N_2 \pi R^2}{l_1 l_2}[/tex]

Pero no cierran las unidades :/

O sea: pienso que no es lo mismo concatenar flujo en un alambre bien ajustado que uno bien estirado, pero no me termino de convencer.

EDIT2: Otra aproximación posible es si se considera que el flujo del solenoide mas corto está confinado él (no escapa el "tubo") entonces la inductancia mutua sería tomando la longitud mas corta.


EDIT: Ahora que lo pienso, por ahí tiene sentido pensando como que hay acoplamiento perfecto y usar [tex]M = \sqrt{L_1 L_2}[/tex]. Pero sin un núcleo ferromagnético sería una aproximación muy pedorra. Quedaría:

[tex]M = \frac{\mu_0 N_1 N_2 \pi R^2}{\sqrt{l_1 l_2}}[/tex]


   OcultoGalería Personal de koreanoVer perfil de usuarioEnviar mensaje privado
Jackson666
Nivel 9


Edad: 37
Registrado: 01 Feb 2009
Mensajes: 1980
Ubicación: Martínez
Carrera: Electricista
CARRERA.electrica.3.jpg
MensajePublicado: Jue Nov 24, 2011 7:12 pm  Asunto:  (Sin Asunto) Responder citandoFin de la PáginaVolver arriba

Che, tengo una duda medio (bastante) boluda. Y tiene que ver más con matemática que con física.

Tengo una bobina arrollada sobre un toroide magnético (una rosca de pascuas) de sección circular. Conozco [tex]i(t)[/tex] (que circula por la bobina y varía armónicamente en el tiempo), [tex]\mu_{r}[/tex], [tex]N[/tex] y todos los datos que hagan falta saber. Quiero calcular el coeficiente de autoinducción L, calculando el flujo de B a través de la sección circular del toroide y derivándolo respecto de la corriente.

De la ley de Ampère sale que [tex]\vec{B}(r,t) = \frac{\mu_{0}\mu_{r}Ni(t)}{2\pi r} \; \hat{\varphi}[/tex]. Para hacer este cálculo se consideró un sistema de coordenadas [tex](r, \varphi, z)[/tex] en donde el toroide (visto de arriba) está en el plano [tex]r\varphi[/tex].

Cuando quiero calcular el flujo [tex]\Phi = \iint_{S}{\vec{B} \cdot d\vec{S}}[/tex], tengo dudas en cómo se expresa [tex]d\vec{S}[/tex] para el caso de sección circular (para el caso cuadrado es fácil). Sé que va a ser [tex]d\vec{S} = dS \; \hat{\varphi}[/tex], pero... ¿Cómo es la expresión correcta de [tex]dS[/tex] (el diferencial escalar de superficie)?.

La sección circular del toroide está en el plano [tex]rz[/tex]. Entonces, ¿es [tex]dS = r dr d\theta[/tex]? ¿O es [tex]dS = drdz[/tex]?. Si fuera el primero lo entiendo, si fuera el segundo no llego a ver por qué. Porque al ser un círculo la sección transversal, debería barrer tanto en radio como en ángulo, ¿verdad?.

Gracias Very Happy.


Aries Género:Masculino Gato OfflineGalería Personal de Jackson666Ver perfil de usuarioEnviar mensaje privado
koreano
Nivel 9



Registrado: 15 Jul 2010
Mensajes: 1796

Carrera: No especificada
blank.gif
MensajePublicado: Jue Nov 24, 2011 10:04 pm  Asunto:  (Sin Asunto) Responder citandoFin de la PáginaVolver arriba

En el sistema de coordenadas en el que estás, sería [tex]dS = drdz[/tex] pero tus límites de integración serían una poronga con raíces cuadradas para una sección circular. Acá tenés el caso de sección cuadrada, fijate que dr y dz son independientes en términos de los límites de integración:

Image

(Sacado de acá, 25:00 por ahí)


Pensá que la descripción de la frontera sección si es circular, en las coordenadas r y z es:

[tex]z^2 + \left(r - \left(\frac{b-a}{2}+a\right)\right)^2 = \left( \frac{b-a}{2} \right)^2[/tex]

Siendo a y b el radio interior y exterior respectivamente. Entonces para integrar podés hacer por ejemplo [tex]z(r)[/tex], integrás primero r y después en z, poniendo como límites de integración para z [tex]\pm \sqrt{\left( \frac{b-a}{2} \right)^2 - \left(r - \left(\frac{b-a}{2}+a\right)\right)^2}[/tex]


La otra opción es hacer una parametrización de la sección usando trigonometría, sacar el Jacobiano y va a salir algo parecido a lo que pusiste como primera opción.


   OcultoGalería Personal de koreanoVer perfil de usuarioEnviar mensaje privado
Jackson666
Nivel 9


Edad: 37
Registrado: 01 Feb 2009
Mensajes: 1980
Ubicación: Martínez
Carrera: Electricista
CARRERA.electrica.3.jpg
MensajePublicado: Jue Nov 24, 2011 10:14 pm  Asunto:  (Sin Asunto) Responder citandoFin de la PáginaVolver arriba

Sí, sí. El de sección cuadrada es sencillo. Lo que había pensado para este es hacer un cambio de coordenadas otra vez, o sea, usar cilíndricas de nuevo por ejemplo. De hecho lo hice, pero llego a que el flujo da 0, lo que no tiene sentido y no entiendo qué es lo que hago mal.

Yo llegué a lo mismo que pusiste (lo de las raíces), pero pensé que había una forma más "elegante". Por ahí la hay y no la estoy viendo :P. De todas formas gracias Smile.


Aries Género:Masculino Gato OfflineGalería Personal de Jackson666Ver perfil de usuarioEnviar mensaje privado
Huey 7
Nivel 6



Registrado: 03 Mar 2010
Mensajes: 267

Carrera: Electrónica
CARRERA.electronica.5.gif
MensajePublicado: Vie Nov 25, 2011 9:16 pm  Asunto:  (Sin Asunto) Responder citandoFin de la PáginaVolver arriba

Jackson666 escribió:
Sí, sí. El de sección cuadrada es sencillo. Lo que había pensado para este es hacer un cambio de coordenadas otra vez, o sea, usar cilíndricas de nuevo por ejemplo. De hecho lo hice, pero llego a que el flujo da 0, lo que no tiene sentido y no entiendo qué es lo que hago mal.

Yo llegué a lo mismo que pusiste (lo de las raíces), pero pensé que había una forma más "elegante". Por ahí la hay y no la estoy viendo :P.

En coordenadas cilíndricas [tex]\textstyle \rho \mbox{, } \varphi \mbox{ y } z[/tex], si R es (b + a)/2, o sea, el radio del toroide, y c es (b - a)/2, o sea, el radio de su sección, los puntos del toroide satisfacen:
[tex](\rho - R)^2 + z^2 \leq c^2.[/tex]

Haciendo en cambio de variables:

[tex]\left \{ \begin{array}{l}\rho = R + r \sen \theta \\z = r \cos \theta\end{array} \right .[/tex]

con [tex]\textstyle r \geq 0 \wedge 0 \leq \theta \leq 2 \pi[/tex], esto se convierte en:
[tex]r^2 \sen^2 \theta + r^2 \cos^2 \theta = r^2 \leq c^2[/tex]

Podríamos decir que [tex]\textstyle r \mbox{, } \theta \mbox{ y } \varphi[/tex] serían "coordenadas toroidales", y cumplen las relaciones:

[tex]\left \{ \begin{array}{l}x = \rho \cos \varphi = (R + r \sen \theta) \cos \varphi \\y = \rho \sen \varphi = (R + r \sen \theta) \sen \varphi \\z = r \cos \theta\end{array} \right .[/tex]

Cualquier sección del toroide tiene [tex]\textstyle \varphi[/tex] constante para todos los puntos, y para calcular una integral se necesita el producto vectorial de las derivadas parciales respecto de r y [tex]\textstyle \theta[/tex]:

[tex](\sen \theta \cos \varphi \hat i + \sen \theta \sen \varphi \hat j + \cos \theta \hat k) \times(r\cos \theta \cos \varphi \hat i + r\cos \theta \sen \varphi \hat j - r\sen \theta \hat k) =[/tex]
[tex]= -r \sen \varphi \hat i + r \cos \varphi \hat j = r \hat \varphi[/tex]

En coordenadas cilíndricas, [tex]\textstyle \vec B[/tex] es:
[tex]\vec B(\rho, \varphi, t) = \frac{\mu N I(t) \hat \varphi}{2 \pi \rho}[/tex]

pasando a "coordenadas toroidales":
[tex]\vec B(r, \theta, \varphi, t) = \frac{\mu N I(t) \hat \varphi}{2 \pi (R + r \sen \theta)}[/tex]

Y el flujo magnético en el sentido de los [tex]\textstyle \varphi[/tex] crecientes (normal [tex]\textstyle \hat \varphi[/tex]) es:

[tex]\Phi(t) = \int^c_0 \left ( \int^{2\pi}_0 \vec B \bullet r \hat \varphi d\theta \right ) dr =\frac{\mu N I(t)}{2\pi} \int^c_0 \left ( \int^{2\pi}_0 \frac{rd\theta}{R + r \sen \theta} \right ) dr[/tex]

No me gustan los diferenciales "sueltos", pero esto vendría a significar que [tex]\textstyle \overrightarrow{dS} = r \hat \varphi drd\theta[/tex].

_________________
Comisión de Estudiantes de Ingeniería Electrónica (ComElec)
Lista de correo - Página Web - Facebook

 Género:Masculino  OfflineGalería Personal de Huey 7Ver perfil de usuarioEnviar mensaje privado
koreano
Nivel 9



Registrado: 15 Jul 2010
Mensajes: 1796

Carrera: No especificada
blank.gif
MensajePublicado: Sab Nov 26, 2011 12:55 pm  Asunto:  (Sin Asunto) Responder citandoFin de la PáginaVolver arriba

Aplauso

Para leer un poco mas: http://en.wikipedia.org/wiki/Toroidal_coordinates


   OcultoGalería Personal de koreanoVer perfil de usuarioEnviar mensaje privado
Jackson666
Nivel 9


Edad: 37
Registrado: 01 Feb 2009
Mensajes: 1980
Ubicación: Martínez
Carrera: Electricista
CARRERA.electrica.3.jpg
MensajePublicado: Sab Nov 26, 2011 1:13 pm  Asunto:  (Sin Asunto) Responder citandoFin de la PáginaVolver arriba

Muy bueno Huey 7! Muchas graciasssss!! Very Happy


Aries Género:Masculino Gato OfflineGalería Personal de Jackson666Ver perfil de usuarioEnviar mensaje privado
Huey 7
Nivel 6



Registrado: 03 Mar 2010
Mensajes: 267

Carrera: Electrónica
CARRERA.electronica.5.gif
MensajePublicado: Sab Nov 26, 2011 4:45 pm  Asunto:  (Sin Asunto) Responder citandoFin de la PáginaVolver arriba

koreano escribió:
Para leer un poco mas: http://en.wikipedia.org/wiki/Toroidal_coordinates

Ja, ja, mirá vos qué interesante, no sabía que existía algo así.

Espero que quede claro que no es eso lo que usé en el cálculo del flujo, no tengo tanta imaginación Jajaja.Dije "coordenadas toroidales" (con comillas) para referirme al cambio de variables que usé con algún nombre, y solamente porque la superficie del toroide con ese cambio de variables particular tiene la expresión sencilla r = c.

_________________
Comisión de Estudiantes de Ingeniería Electrónica (ComElec)
Lista de correo - Página Web - Facebook

 Género:Masculino  OfflineGalería Personal de Huey 7Ver perfil de usuarioEnviar mensaje privado
Hermitico
Nivel 4


Edad: 32
Registrado: 19 Nov 2011
Mensajes: 68

Carrera: Industrial
argentina.gif
MensajePublicado: Mar Nov 29, 2011 10:45 pm  Asunto:  (Sin Asunto) Responder citandoFin de la PáginaVolver arriba

http://wiki.foros-fiuba.com.ar/materias:62:03:parcial_20070630

En el punto 6, tenes una barra que se mueve con velocidad v. Y un hilo infinito por el cual pasa una corriente I. El hilo genera un B que se puede averiguar por Ampere. El tema es la disposición de la barra. En diferentes puntos de la barra tiene distinto valor de B. Si me dieran el ancho de la barra podía calcular por Faraday la variación del flujo con la integral doble, pero no me lo dan. Alguna idea?


Leo Género:Masculino Cabra OfflineGalería Personal de HermiticoVer perfil de usuarioEnviar mensaje privado
koreano
Nivel 9



Registrado: 15 Jul 2010
Mensajes: 1796

Carrera: No especificada
blank.gif
MensajePublicado: Mar Nov 29, 2011 10:56 pm  Asunto:  (Sin Asunto) Responder citandoFin de la PáginaVolver arriba

Hacé lo mismo que la solución del problema de la barra en esta guía (4a), que utiliza un diferencial de tiempo para considerar a la barra cerrando el circuito con sí misma. Una vez que tenés un área con extension podés integrar para conocer el flujo total del campo B no uniforme.

No sé si ese tipo de posts pertenece a este topic though. Probablemente estaría bueno que se separen a otro.


   OcultoGalería Personal de koreanoVer perfil de usuarioEnviar mensaje privado
Hermitico
Nivel 4


Edad: 32
Registrado: 19 Nov 2011
Mensajes: 68

Carrera: Industrial
argentina.gif
MensajePublicado: Mar Nov 29, 2011 11:20 pm  Asunto:  (Sin Asunto) Responder citandoFin de la PáginaVolver arriba

Buenisimo, gracias. Igual genera el debate eterno de si se puede o no cerrar imaginariamente el circuito que ya dio para discutir largo y tendido. Espero que lo consideren bien en un parcial.
Con respecto a moverlo de topic, sinceramente nose como hacerlo


Leo Género:Masculino Cabra OfflineGalería Personal de HermiticoVer perfil de usuarioEnviar mensaje privado
connor
Nivel 8


Edad: 38
Registrado: 30 Ene 2010
Mensajes: 620

Carrera: Electrónica
CARRERA.electronica.6.gif
MensajePublicado: Mie Nov 30, 2011 2:12 am  Asunto:  (Sin Asunto) Responder citandoFin de la PáginaVolver arriba

de poder cerrar si se puede, el problema es que para "ciertos profesores" piensan que los alumnos cierran por cerrar sin justificar porque esta bien ese metodo, no vi como resolvieron en este post, pero leone por ejemplo es muy fino en este detalle (y al menos cuando la curse en su momento dijo que no se podia hacer y termino la discusión ), pero hya una justificacion y se puede hacer lo mas bien, de hecho varias cosas te las crees en fisica 2 y no se justifica, esta es una de esas creo

_________________
[tex] \phi (\overrightarrow r ) = \int\limits_V {{d^3}\overrightarrow {r'} \;G(\overrightarrow r ,\overrightarrow {r'} )} \;\rho (\overrightarrow {r'} ) - \frac{1}{{4\pi }}\oint\limits_S {{d^2}\overrightarrow {r'} } \;\frac{{\partial G(\overrightarrow r ,\overrightarrow {r'} )}}{{\partial \overrightarrow {n'} }}\;\phi '(\overrightarrow {r'} ) [/tex]

Sagitario Género:Masculino Bufalo OfflineGalería Personal de connorVer perfil de usuarioEnviar mensaje privado
connor
Nivel 8


Edad: 38
Registrado: 30 Ene 2010
Mensajes: 620

Carrera: Electrónica
CARRERA.electronica.6.gif
MensajePublicado: Mie Nov 30, 2011 2:32 am  Asunto:  (Sin Asunto) Responder citandoFin de la PáginaVolver arriba

koreano escribió:
Este es el primer problema en el que tenemos que apelar a esta fórmula, pero es simple el razonamiento. Recordemos que el rotor de un campo vectorial es un vector, cuya magnitud nos daba una idea de que tanto era rotacional/solenoidal el campo. Sin embargo, es un vector. Y como bien sabemos, ya que viene de un producto vectorial (sort of), su dirección es justamente el eje de dicha rotación que representa. Entonces, qué significa que el rotor de E sea proporcional a la variación de B en este problema? Como vemos en el dibujo, el campo B es entrante y por lo tanto su derivada también es entrante en el plano. Por lo tanto, no queda otra que el campo E siempre que sea generado por inducción sea solenoidal en el plano.


Ojo este razonamiento aca, en principio solenoidal es que el campo tenga divergencia nula en todo punto del espacio, el rotor no me da informacion de eso, solo si es irrotacional.

Pero esto que nombras de que la direccion viene de ver que es un producto vectorial esta mal, en principio porque nabla es un operador y no un vector, no tiene direccion ni sentido ni magnitud, asi que es imposible verlo como producto vectorial.

Lo que se hace es usar esa ecuación, por ejemplo en coordenadas cilindricas (en cartesianas se complica describir), supones que la derivada temporar de B esta en versor z, entonces en la ecuacion del rotor sobrevive solo esa coordenada, de ahi tenes que suponer que no puede tener en r versor y asi podes quedarte con la que te interesa.

No recuerdo si este tipo de análisis corresponde a la materia, pero es una de las tantas demostraciones formales "simples" para el tema, hay que averiguar, pero cuidado con esas demostraciones

_________________
[tex] \phi (\overrightarrow r ) = \int\limits_V {{d^3}\overrightarrow {r'} \;G(\overrightarrow r ,\overrightarrow {r'} )} \;\rho (\overrightarrow {r'} ) - \frac{1}{{4\pi }}\oint\limits_S {{d^2}\overrightarrow {r'} } \;\frac{{\partial G(\overrightarrow r ,\overrightarrow {r'} )}}{{\partial \overrightarrow {n'} }}\;\phi '(\overrightarrow {r'} ) [/tex]

Sagitario Género:Masculino Bufalo OfflineGalería Personal de connorVer perfil de usuarioEnviar mensaje privado
Jackson666
Nivel 9


Edad: 37
Registrado: 01 Feb 2009
Mensajes: 1980
Ubicación: Martínez
Carrera: Electricista
CARRERA.electrica.3.jpg
MensajePublicado: Mie Nov 30, 2011 12:10 pm  Asunto:  (Sin Asunto) Responder citandoFin de la PáginaVolver arriba

Me parece más sencillo pensar que el vector resultante de la cuenta [tex]\vec{\nabla}\times\vec{E}[/tex] tiene que ser ortogonal a [tex]\vec{E}[/tex](*). Si [tex]\vec{\nabla}\times\vec{E} = -\frac{\partial \vec{B}}{\partial t}[/tex] y la derivada está en una sola dirección, a las líneas de [tex]\vec{E}[/tex] no les queda otra que estar contenidas en un plano que tenga como normal a esa dirección.

Sino más sencillo aún. Sabiendo que se puede inferir la dirección de las líneas de inducción magnética mediante la igualdad [tex]\vec{\nabla}\times\vec{B} = \mu_{0} \vec{J}[/tex], se puede ver que la parte funcional de esta ecuación es análoga a la anterior, sólo cambian los actores. Si a través de esta se conocen las líneas de [tex]\vec{B}[/tex], por analogía se conocen las líneas de [tex]\vec{E}[/tex].

(*) De hecho, este producto es ortogonal al operador nabla también, ya que [tex]\vec{\nabla}\cdot\left(\vec{\nabla}\times\vec{E}\right) = \text{div}\left[ \text{rot}\left( \vec{E} \right)\right] = 0[/tex]. Respecto a lo de que "nabla no es un vector, es un operador...", no es 100% así. Su norma está definida sobre espacios de Banach. Si [tex]X[/tex] e [tex]Y[/tex] son espacios normados y [tex]T \in L(X,Y)[/tex] (T es lineal y va de X a Y), se define su norma (por ejemplo) mediante [tex]\left\| T \right\| := \sup\left\{ \left\| T(x) \right\|: \; x \in X,\, \left\| x \right\| < 1\right\}[/tex].

También se puede ver como que [tex]\frac{\partial}{\partial x_{i}}[/tex] representa la derivada direccional respecto del i-ésimo versor canónico; por lo tanto, tendrá esa dirección. Si aún quedan dudas, invito a ver este link.


Aries Género:Masculino Gato OfflineGalería Personal de Jackson666Ver perfil de usuarioEnviar mensaje privado
connor
Nivel 8


Edad: 38
Registrado: 30 Ene 2010
Mensajes: 620

Carrera: Electrónica
CARRERA.electronica.6.gif
MensajePublicado: Mie Nov 30, 2011 1:52 pm  Asunto:  (Sin Asunto) Responder citandoFin de la PáginaVolver arriba

Nos estamos yendo de la materia, pero en electromagnetismo dejamos definido nuiestro espacio vectorial, y no sobre una aplicacion de otro, pero abrimos otro post y te cuento, esto que planteo como error se remarca constantemente en electromagnetismo, de hecho en exactas solo te dejan dos formas de calcular, uno como lo conte y otro por las propiedades del gauge de la fuerza electromagnetica, pero suponer que las lineas son como son por parecerse a un producto vectorial deberia estar mal, el problema es que en fisica 2 son casos simples donde coinciden, pero no esta bien

_________________
[tex] \phi (\overrightarrow r ) = \int\limits_V {{d^3}\overrightarrow {r'} \;G(\overrightarrow r ,\overrightarrow {r'} )} \;\rho (\overrightarrow {r'} ) - \frac{1}{{4\pi }}\oint\limits_S {{d^2}\overrightarrow {r'} } \;\frac{{\partial G(\overrightarrow r ,\overrightarrow {r'} )}}{{\partial \overrightarrow {n'} }}\;\phi '(\overrightarrow {r'} ) [/tex]

Sagitario Género:Masculino Bufalo OfflineGalería Personal de connorVer perfil de usuarioEnviar mensaje privado
Mostrar mensajes de anteriores:      
Responder al tema Ver tema anteriorEnviar por mail a un amigo.Mostrar una Lista de los Usuarios que vieron este TemaGuardar este Tema como un archivoPrintable versionEntrá para ver tus mensajes privadosVer tema siguiente

Ver tema siguiente
Ver tema anterior
Podés publicar nuevos temas en este foro
No podés responder a temas en este foro
No podés editar tus mensajes en este foro
No podés borrar tus mensajes en este foro
No podés votar en encuestas en este foro
No Podéspostear archivos en este foro
No Podés bajar archivos de este foro


Todas las horas son ART, ARST (GMT - 3, GMT - 2 Horas)
Protected by CBACK CrackerTracker
365 Attacks blocked.

Powered by phpBB2 Plus, phpBB Styles and Kostenloses Forum based on phpBB © 2001/6 phpBB Group :: FI Theme :: Mods y Créditos

Foros-FIUBA está hosteado en Neolo.com Cloud Hosting

[ Tiempo: 0.4222s ][ Pedidos: 20 (0.2809s) ]